Dizel Motorun İzentropik İşleminin İşini Belirleme Yöntemleri


2

Bir Dizel Motorun silindirindeki hava, $ 30 ^ o \ $$ C $ ve 138 $ \ text {kPa} $ 'dır. Orijinal hacminin 1 / 18'ine kadar sıkıştırılmışsa. Aşağıdakilerden hangisi silindirin yer değiştirme hacminin sıkıştırılmasında yapılan işe en çok eşdeğerdir, $ 14.2 $ Litre

Cevap $ 11 \ \ text {kJ} $

$$ T_1 = 30 ^ o + C + 273.15 = 303,15 \ K $$ $$ P_1 = 138 \ kPa $$ $$ V_d = 14.2 \ L * (1 \ m ^ 3 / 1000L) = 0,0142 \ m ^ 3 $$

Girişimi 1: İzentropik ilişki kullanma

$$ r_k = 18 = V_1 / V_2 $$ $$ V_1 = \ kırık {mRT} {P_2} = \ kırık {1 * 0.287 * 303.15) {138} = 0.63046 \ m ^ 3 $$ $$ V2 = 0.03502578502 \ m ^ 3 $$ $$ P2 = \ kırılma {mRT} {V2} = \ kırılma {287 * 303.15} {0,0350257) = 2484 \ Pa $$ hava için: $$ k = 1.4 $$ $$ W = \ kırılma {(P_2V_2 - P_1V_1)} {k - 1} = \ metin {(Çok çok küçük bir değer)} $$


Girişimi 2: $ r_k $ ve yer değiştirme hacmini kullanarak:

$$ r_k = \ kırılma {(V_d + V_c)} {V_c} $$ $$ V_d = 14.2L = 0.0142 \ m ^ 3 $$ $$ V_c = 8.3529411 * 10 ^ {- 4} = V_2 $$ $$ V_1 = V_c + V_d = 0.01583529 \ m ^ 3 $$ $$ P_1V_1 ^ k = P_2V_2 ^ k $$ $$ \ frak {138 * 0.01583529 ^ {1.4}} {(8.3529411 * 10 ^ {- 4}) ^ {1.4}} = P2 = 8487.512 \ kPa $$

Her şeyi iş işlevine takın $$ W = \ kırılma {P_2V_2 - P_1V_1} {k-1} = 12.1 \ kJ $$

Tamam, yaklaşıyorum ama neyi yanlış yaptığımı bilmiyorum ... Herhangi bir ipucu? Ve neden 'yöntem 1 çalışmıyor?

Yanıtlar:


2

İkinci girişiminiz doğru, iki hata yaptınız:

  1. $ V_1 $ 'ı tekrar hesapladıysanız, 0,01503 $ 0,0158 $ $ m ^ 3 $.

  2. $ P_2 $ ile aynı sorun. $$ P_2 = \ kırılma {138 * 10 ^ {3} * 0.01503 ^ {1.4}} {(8.3529411 * 10 ^ {- 4}) ^ {1.4}} = 7.889 * 10 ^ 6 \ Pa $$

İzentropik iş denkleminde yer değiştirme: $$ W = \ kırık {P_2V_2 - P_1V_1} {k-1} = \ kırık {(7.867 * 10 ^ 6 * 8.3529411 * 10 ^ {- 4}) - (138 * 10 ^ 3 * 0.01503)} 1} = 11.2 \ kJ $$

Hangi problemin cevabına çok yakın, hangi cevabı sorduğuna göre çoktan seçmeli bir soru olduğunu düşünüyorum. neredeyse eşittir .


1

Aşağıdakilerin "yoğun" sorunu çözmek için zaten yeterli olduğunu düşünün:

Dizel Motorun silindirindeki hava $ 30 \, {} ^ o \ mathrm {C} $ ve 138 $, \ mathrm {kPa} $ 'dır. Daha sonra $ 1/18 \ 'e kadar sıkıştırılmışsa, $ $ orijinal hacmi, sistemde yapılan işi hesaplayın.

sıkıştırma oranı $ \ varrho = v_1 / v_2 = 18 $ 'dır ve barometrik oran $ \ beta = P_2 / P_1 $ izoroprop ilişkisinden hızlı bir şekilde türetilir ($ \ gama = 1.4 $ sıcaklıktan bağımsız olduğu için): $$ P_1v_1 ^ \ gama = P_2v_2 ^ \ gama \ - \ frak {P_2} {P_1} = \ left (\ frak {v_1} {v_2} \ sağ) ^ \ gama \ - \ beta = \ varrho ^ \ gama $$ bu doğrudan iş denkleminde kullanılabilir. Sonuncusu, aşağıda gösterildiği gibi, yoğun ilişki kullanılarak, Clapeyron Diyagramında (kapalı bir sistemde izoropropik şaft çalışması) $ P \, \ mathrm {d} v $ integralinden türetilmiştir: \ Begin {align}  W = - \ int_ {v_1} ^ {V_2} p \, \ mathrm {d} v & Sons = - \ int_ {v_1} ^ {V_2} \ frac {P_1v_1 ^ {\ y}} {v ^ \ y} \ , \ mathrm {d} v \\ & Sons = P_1v_1 ^ {\ y} \ int_ {v_1} ^ {V_2} v ^ {- \ y} \, \ mathrm {d} v \\ & Sons = P_1v_1 ^ {\ y} \ frac {1} {\ gamma-1} \ sol (V_2 ^ {gama \ 1-} doğru -v_1 ^ {gama \ 1-} \) \\ & Sons = RT_1 \ frac {1} {\ gamma-1} \ sol (V_2 ^ {gama \ 1-} / v_1 ^ {gama \ 1-} -1 \ sağ) \\ & Sons = RT_1 \ frac {1} {\ gamma-1} \ sol (\ varrho ^ {\ gamma-1} -1 \ sağ) \ Ucu {hizalama}

Sonuç, yoğun bir miktarı tanımlayacağından, kapsamlı sonuç, sistemde bulunan (ve kullanılmayan) kimyasal maddeye bağlı olacaktır. Bu, yer değiştiren birimden ve sıkıştırma oranından türetilir, yani terimleri $ \ varrho $ olarak vurgulayarak yeniden düzenlenir:

Silindirin deplasman hacmine $ \ Delta V = 14.2 \, \ mathrm

$$ \ Delta V = V_1-V_2 \ ila \ frak {\ Delta V} {V_2} = \ frak {V_1-V_2} {V_2} = \ varrho-1 \ - V_2 = \ frak {\ Delta V} {\ varrho- 1} $$ buradan itibaren ilk cilt bilinmektedir ($ V_2 $ terim yerine): $$ V_1 = V_2 + \ Delta V = \ frak {\ Delta V} {\ varrho-1} + \ Delta V = \ frak {14.2 \, \ mathrm {L}} {17} +14.2 \, \ mathrm {L} = 15.03 \, \ mathrm {L} = 15.03 \ times10 ^ {- 3} \, \ mathrm {m ^ 3} $$ İdeal Gaz EoS içindeki tüm ilk yoğun parametrelerin kullanılmasıyla sabit molar hava miktarı bilinmektedir: $$ n = \ frac {P_1V_1} {RT_1} = \ frac {15.03 \ times10 ^ {- 3} \, \ mathrm {m ^ 3} \ cdot138 \ times10 ^ {3} \, \ mathrm {Pa}} {8.314 \ (\ mathrm {Pa \ m ^ 3 \, mol ^ {1 -} \ K ^ {- 1}}) \ cdot303.15 \, \ mathrm {K}} = 0.823 \, \ mathrm {mol} $$ $ W $ kapsamlı parametresinin $ nw $ olarak ve önceki denklemde $ w = 13.72 \, \ mathrm {kJ \, mol ^ {- 1}} $ $ olduğunu bilerek, istenen değer $ W = 0.823 \ , \ mathrm {mol} \ cdot 13.72 \, \ mathrm {kJ \, mol ^ {- 1}} = 11.29 \, \ mathrm {kJ} $.

Bu noktada, sonuç ilk verenden farklı olsa bile, bir açıklama olabilir. İlk yönteminizde, başlangıçtaki hava kütlesini $ 1 \, \ mathrm {kg} $ 'a eşit olarak kabul ettiğiniz için, bu, Devlet Denklemine aykırıdır. Başlangıçtaki (ve sabit) molar hava miktarı, başlangıçtaki geniş kapsamlı $ (T_1, P_1, V_1) $ durumuna bağlı olduğundan, yer değiştirme hacmi ve sıkıştırma oranı biliniyorsa, sadece $ V_1 $ bırakılan tek değişken elde edilir.


Harika iş Andrea, ama neden yoğun mekanik işi hesapladığınızı $ w = \ int_ {P_1} ^ {P_2} v \, \ mathrm {d} P $ ile izentropik sabit akışlı şaft çalışması olarak (burada kapalı bir sistem var) ve $ w = \ int_ {v_1} ^ {v_2} P \, \ mathrm {d} v $ sınır çalışması olarak değil mi?
Algo

@Algo Benim akıl yürütme, sabit olmayan bir kontrol hacminin içindeki yoğun diferansiyel ilişkinin $ T \ mathrm {d} s = \ mathrm {d} hv \ mathrm {d} P $ alacağı varsayımıyla gitti, çünkü burada $ w olarak değerlendirdim entalpi varyasyonu olarak $. Sanki sıkıştırma aşaması (giriş akışının varlığında, gerçek bir motorun içinde) Çalışan bir Makineye eşdeğer olacak gibi
alandella

Ancak bu bizim durumumuza uygulanamaz, bunun tam bir dizel döngüsü olmadığını söylememek değil, sadece basit bir izentropik sıkıştırma işlemidir. Bununla birlikte, idealize edilmiş basit güç döngüsü ile uğraşırken genellikle tüm genişletme ve sıkıştırma işlemlerinin yarı denge şeklinde gerçekleştiğini varsayıyoruz.
Algo

Neyse, $ p \, dv $ asla ihmal edilmez physics.stackexchange.com/questions/52001/...
Algo

@Algo Burada değiştirdim ve böylece düzelttim. Hatayı fark ettiğiniz için tekrar teşekkür ederiz;)
alandella
Sitemizi kullandığınızda şunları okuyup anladığınızı kabul etmiş olursunuz: Çerez Politikası ve Gizlilik Politikası.
Licensed under cc by-sa 3.0 with attribution required.